LSAT and Law School Admissions Forum

Get expert LSAT preparation and law school admissions advice from PowerScore Test Preparation.

 Administrator
PowerScore Staff
  • PowerScore Staff
  • Posts: 8916
  • Joined: Feb 02, 2011
|
#33528
Complete Question Explanation

Justify the Conclusion—SN. The correct answer choice is (A)

In this example, the author presents several conditional statements and draws a conclusion that requires further justification. Since the conclusion appears in the middle of the stimulus, it might be helpful to consider the author’s argument slightly reordered:

In the first sentence of the stimulus, the author provides if the book that Skiff has written is published this year, Professor Nguyen will urge the dean to promote Skiff:
  • Premise (1): ..... Skiff book published ..... :arrow: ..... Prof will urge for promotion
In the last sentence of the stimulus, the author adds that the dean would surely heed such a recommendation from Professor Nguyen:
  • Premise (2): ..... Skiff book published ..... :arrow: ..... Prof urge for promotion ..... :arrow: ..... Skiff promoted
Thus, the author concludes, if the book is as well-written as Skiff claims, Skiff will be promoted:
  • Conclusion: ..... Book important, well-written as claimed ..... :arrow: ..... Skiff promoted
The question stem asks us to identify a statement that, if assumed, would allow the conclusion to be properly drawn. Despite the word “assumed” in the stem, this is a Justify question because our job is not to identify a statement upon which the argument depends, but rather to prove the conclusion by adding a piece of information to the premises. The sufficient condition indicator (“if”) in the question stem is a reminder that you must select an answer that is sufficient to prove the conclusion by using the Justify Formula:
  • Premises + Answer choice = Conclusion

    Premises: ..... Skiff book published ..... :arrow: ..... Prof urge for promotion ..... :arrow: ..... Skiff promoted

    Justify Answer: ..... ?

    Justified Conclusion : ..... Book important, well-written as claimed ..... :arrow: ..... Skiff promoted
Note that Skiff’s promotion is an element that is common to the premises and the conclusion. The correct answer choice will likely tie together the other, rogue elements of the argument.

Answer choice (A): This is the correct answer choice, as this choice provides the assumption which, when added to the stimulus’ premises, fully justifies the author’s conclusion:
  • Premise: ..... Skiff book published ..... :arrow: ..... Prof N urge for promotion ..... :arrow: ..... Skiff promoted

    Choice (A): ..... Book important, well written as claimed ..... :arrow: ..... Skiff book published

    Conclusion: ..... Book important, well-written ..... :arrow: ..... Skiff promoted
Answer choice (B): This answer choice is a Mistaken Reversal of the second premise:
  • Premise: ..... Skiff book published ..... :arrow: ..... Skiff promoted

    Choice (B): ..... Skiff promoted ..... :arrow: ..... Skiff book published
Answer choice (C): The professor’s belief is irrelevant to the author’s conclusion. This answer choice is incorrect.

Answer choice (D): This answer choice may seem attractive, but upon closer inspection it should be clear that the conditional relationship outlined here cannot justify the conclusion:
  • Premise: ..... Skiff book published ..... :arrow: ..... Prof N urge for promotion ..... :arrow: ..... Skiff promoted

    Choice (D): ..... Skiff book published ..... :arrow: ..... Book important, well written as claimed

    Justified Conclusion: ..... ?
Both the premise chain and answer choice (D) posit the publishing of Skiff’s book as a sufficient condition, making it impossible to link them in a conditional chain.

Answer choice (E): This answer choice is a Mistaken Reversal of the second premise:
  • Premise: ..... Prof urge for promotion ..... :arrow: ..... Skiff promoted

    Choice (E): ..... Skiff promoted ..... :arrow: ..... Prof urge for promotion
 Lampostman
  • Posts: 1
  • Joined: Sep 22, 2014
|
#16866
Hello.

I'm having a bit of a problem with this question.

"If Skiff's book is Published this year, Professor Nguyen vows she will urge the dean to promote skiff. Thus, if Skiffs book is as important and as well written as Skiff claims, he will be promoted, for Nguyen will certainly keep her promise, and the dean will surely promote Skiff if Nguyen recommends it"


After reviewing the answer choices, I was going to immediately select A - Skiff's book will be published this year if it is as important as he claim it is. However, it seemed like a trick to me. Is it not possible that Skiff's book is important, but not well written, and will therefore not be published.

Thank you
 Nikki Siclunov
PowerScore Staff
  • PowerScore Staff
  • Posts: 1362
  • Joined: Aug 02, 2011
|
#16875
Hi Lampostman,

Thanks for your question. Let's break down the argument first. Since the conclusion appears in the middle of the stimulus, it might be helpful to consider the author’s argument slightly reordered:

In the first sentence of the stimulus, the author provides if the book that Skiff has written is published this year, Professor Nguyen will urge the dean to promote Skiff:

Skiff book published :arrow: Prof will urge for promotion

In the last sentence of the stimulus, the author adds that the dean would surely heed such a recommendation from Professor Nguyen:

Skiff book published :arrow: Prof will urge for promotion :arrow: Skiff promoted

Thus, the author concludes, if the book is as well-written as Skiff claims, Skiff will be promoted:

Book important, well-written as claimed :arrow: Skiff promoted

As you state, answer choice (A) immediately justifies the conclusion, because it provides the assumption which, when added to the stimulus’ premises, fully justifies the author’s conclusion. It is not necessary that answer choice (A) identify both of the qualities mentioned in the conclusion. Remember - this is a Justify question: the correct answer choice, if true, must prove the conclusion. Thus, if the book's proclaimed importance is sufficient to establish that the book will be published, the conclusion is automatically justified regardless of whether Skiff's book is well-written.

Does that make sense?
 Haleyeastham
  • Posts: 33
  • Joined: Aug 03, 2015
|
#19294
The reason I did not choose answer choice A for this question is because it only said his book was important, not important AND well written like it needs to be for the book to be public.... How is this justified?

I don't to make sure this method of thinking doesn't mess me up going forward.

Thanks!
 Jon Denning
PowerScore Staff
  • PowerScore Staff
  • Posts: 904
  • Joined: Apr 11, 2011
|
#19331
Hi Haley,

Thanks for the question! I'm actually not seeing this as question 21 from either LR section in June 2003. #21 in LR1 is about pizzerias, and in LR2 it's about a university budget.

I thought maybe it was in reference to LR1 question 20, where the correct answer (D) is about a book--a novel that is both critically-acclaimed and widely-read--but that doesn't seem to match up, either. Can you double check your question and let me know? Thanks, and if I've missed something obvious here (I concede it's entirely possible) please accept my apology :)

Jon
 Haleyeastham
  • Posts: 33
  • Joined: Aug 03, 2015
|
#19343
I am so sorry, I meant to say June 2013, not 2003- section 5, number 21.

Thank you!!
 David Boyle
PowerScore Staff
  • PowerScore Staff
  • Posts: 836
  • Joined: Jun 07, 2013
|
#19345
Haleyeastham wrote:I am so sorry, I meant to say June 2013, not 2003- section 5, number 21.

Thank you!!
Hello Haleyeastham,

"The reason I did not choose answer choice A for this question is because it only said his book was important, not important AND well written like it needs to be for the book to be public.... How is this justified?" Good question.
However, if we look at it carefully here, the stimulus doesn't say that the book NEEDS to be important and well written. Important and well written are given as sufficient condition, not necessary conditions. Maybe either one would be sufficient.
With answer A, "Skiff’s book will be published this year if it is as important as he claims it is", that gives a pass around the "well written" part and says that "important" alone will do the trick. "Important" leads to "published", which leads to Nguyen urging promotion, which leads to promotion, from what the stimulus tells us.
So, answer A gets us onto that chain above, leading to promotion. But your question was good.

Hope this helps,
David
 courtneym1991
  • Posts: 2
  • Joined: Sep 03, 2015
|
#19640
Hello!

I'm struggling with the conditional logic in question 21, about Skiff's promotion and book publication. I understand why A is correct, but I don't understand why D is wrong.

I get the logic in the stimulus:

published -> urge promotion -> promoted

important & well written -> promoted

And in answer choice A: important -> published. I see how this connects important -> published -> urge promotion -> promoted.

But why doesn't the same occur in answer choice D? The logic states that: published -> important & well written.

Since "published" and "important and well written" are both sufficient elements in their respective logic chains, why doesn't it work that: published -> important & well written -> promoted?

I would love to learn where I went wrong here. Thanks!

-Courtney
 jeff.wren
PowerScore Staff
  • PowerScore Staff
  • Posts: 26
  • Joined: Jul 04, 2015
|
#19643
Hi Courtney,

Your diagramming for this question looks solid. Just to be clear, the first chain that you have is our premises and the second one represents the conclusion that we are trying to get to.

Premises: published -> urge promotion -> promoted

Conclusion: Important and well written -> promoted

What we need is an answer that links the sufficient part of our conclusion (important and well written) to our premises in the correct way so that we can follow the arrows in the reasoning chain and end up with our necessary term (promoted) in the conclusion.

Answer (A) does this by linking: (Important -> published) in the correct sufficient/necessary order. The fact that this answer only mentions "important" and not "well written" doesn't matter because it still justifies the argument. In other words, if "important" guarantees that it gets published, the fact that it is also well written doesn't matter.

The problem with (D) is that it contains a Mistaken Reversal. Your diagram for this question is correct:

published -> Important and well written

The problem is that this answer doesn't prove our conclusion because this is saying that if the book gets published, then we know it was important and well written, but it is possible that the book was important and well written but still didn't get published. The necessary can occur without the sufficient.

Therefore, adding answer (D) to our diagram doesn't prove our conclusion that:

if important and well written -> promoted

because there is no guarantee that the book was published with this answer.

I hope this helps.

Best,
Jeff
 courtneym1991
  • Posts: 2
  • Joined: Sep 03, 2015
|
#19644
SUPER helpful. Thank you!!!

Get the most out of your LSAT Prep Plus subscription.

Analyze and track your performance with our Testing and Analytics Package.